Is Exponential Form Better for Balloon with Inserted Load Calculation?

  • Thread starter Hak
  • Start date
  • Tags
    Physics
  • #106
Hak said:
From this differential equation, I obtain:

$$\rho(h) = k e^{- \frac{gh}{RT}}$$.

What should I get, comparing with the expression of ##\rho## given in the text?
How about you show some steps?
 
Physics news on Phys.org
  • #107
erobz said:
How about you show some steps?
Yes, I show the steps now, but is this result right?
 
  • #108
Hak said:
Yes, I show the steps now, but is this result right?
I don't know, It's not an explicit result in my book. I would like to see the steps you have taken to solve the equation. If each main step was executed correctly, then the result would be too.
 
Last edited:
  • #109
erobz said:
I don't know, It's not an explicit result in my book. I would like to see the steps you have taken to solve the equation. If each main step was executed correctly, then the result would be too.
OK, I will insert all the steps shortly. In case my result is right, what should I get from comparing the two expressions? In other words, what considerations should I make?
 
  • #110
Hak said:
OK, I will insert all the steps shortly. In case my result is right, what should I get from comparing the two expressions? In other words, what considerations should I make?
One step at a time please. If you show some basic steps how you arrived at the solution ( verifying if it is correct or not- something you are very persistent in having others do in your problems), we can move forward with what it means.
 
Last edited:
  • #111
erobz said:
Yes.

Then search "variation of temperature in troposphere vs altitude".

Then if you want to assume the ideal gas law model use that empirical result for the temperature to solve the hydrostatic differential equation for the density and compare with ##\rho = \rho_o( 1 - \alpha h)##

$$ \frac{dP}{dz} = -\rho g $$
Preface: I am very insistent that others check my work because this is the only way to understand whether my theories and procedures are reasonable or not. I hope I have not annoyed anyone.

Below are the steps I took to calculate the differential equation ## \frac{dP}{dh} = -\rho g \Rightarrow \ RT \frac{d \rho}{dh} = -\rho g##.

This equation can be written as:

$$\rho' + \frac{g}{RT} \rho = 0$$.

It is a first-order linear ordinary differential equation of the type:

$$y'(h) + a_0 (h) y(h) = 0$$, which has solution ## y(h) = k \cdot e^{-A(h)}##, with ##A(h) = \int a_0(h) dh##.

Since, in this case, ##A(h) = \frac{g}{RT} h##, we have:

$$\rho (h) = k \cdot e ^{- \frac{g}{RT} h}$$

What do you think?
 
  • #112
Hak said:
Below are the steps I took to calculate the differential equation ## \frac{dP}{dh} = -\rho g \Rightarrow \ RT \frac{d \rho}{dh} = -\rho g##.
##T## isn't a constant, you can't take it out of the derivative.

$$ \frac{d}{dh} ( \rho T ) = - \frac{\rho g }{R} $$

Continue from here.
 
  • #113
erobz said:
##T## isn't a constant, you can't take it out of the derivative.

$$ \frac{d}{dh} ( \rho T ) = - \frac{\rho g }{R} $$

Continue from here.
You are right. I'm having difficulty solving this ODE, I can't figure out how to work with another dependent variable from ##h##. Do you have any advice?
 
  • #114
Hak said:
You are right. I'm having difficulty solving this ODE, I can't figure out how to work with another dependent variable from ##h##. Do you have any advice?
##T## is a function of ##h##, use the product rule to complete the derivative on the LHS.
 
  • #115
erobz said:
##T## is a function of ##h##, use the product rule to complete the derivative on the LHS.
Following your advice, I arrive at the equation:

$$\rho' T + T' \rho+ \frac{g}{R} \rho = 0 $$, from which:

$$\rho' T + T_0 \beta \rho + \frac{g}{R} \rho = 0 $$. Therefore:

$$\rho' T + \left(T_0 \beta + \frac{g}{R} \right) \rho = 0$$, whence:

$$\rho' + \frac{1}{T} \left(T_0 \beta + \frac{g}{R} \right) \rho = 0$$.

Following the procedure of the previous message, we have:

$$A(h) = \int \frac{1}{T(h)} \left(T_0 \beta + \frac{g}{R} \right) dh = \left(T_0 \beta + \frac{g}{R} \right) \int \frac{1}{T_0(1 + \beta h)} dh = \left(T_0 \beta + \frac{g}{R} \right) \frac{ln |1 + \beta h|}{T_0 \beta} h + c$$. Finally:

$$\rho (h) = k \cdot e ^{-\left(T_0 \beta + \frac{g}{R} \right) \frac{ln |1 + \beta h|}{T_0 \beta } h + c}$$.

What do you think?
 
  • #116
There is a sign error here, note that ##T = T_o - T_o \beta h ##.
Hak said:
$$\rho' T + T_0 \beta \rho + \frac{g}{R} \rho = 0 $$.

Hak said:
$$\rho' + \frac{1}{T} \left(T_0 \beta + \frac{g}{R} \right) \rho = 0$$.
Once you fix the sign error write

$$ \frac{d \rho}{dh} = \cdots $$

Then separate variables and integrate both sides.
 
  • #117
Ok, I'm fixing the sign error.$$\rho' T + T' \rho + \frac{g}{R} \rho = 0 $$, from which:
$$\rho' T - T_0 \beta \rho + \frac{g}{R} \rho = 0 $$. Therefore:
$$\rho' T + \left(\frac{g}{R} - T_0 \beta \right) \rho = 0$$, whence:
$$\rho' + \frac{1}{T} \left(\frac{g}{R} - T_0 \beta \right) \rho = 0$$.
Following the procedure of the previous message, we have:
$$A(h) = \int \frac{1}{T(h)} \left(\frac{g}{R} - T_0 \beta \right) dh = \left(\frac{g}{R} - T_0 \beta \right) \int \frac{1}{T_0(1 - \beta h)} dh = - \left(\frac{g}{R} - T_0 \beta \right) \frac{ln |1 - \beta h|}{T_0 \beta} h + c$$. Finally:
$$\rho (h) = k \cdot e ^{\left(\frac{g}{R} - T_0 \beta \right) \frac{ln |1 - \beta h|}{T_0 \beta } h + c}$$.
Is it really necessary to trace the ODE back to a separable variable differential equation? Is my procedure wrong? I can't do it directly, however...
 
  • #118
Hak said:
Is it really necessary to trace the ODE back to a separable variable differential equation? Is my procedure wrong?
Yes. You are not getting the proper result.
 
  • #119
erobz said:
Yes. You are not getting the proper result.
Okay. I get to the equation:

$$T \ d \rho = \left(T_0 B - \frac{g}{R} \right) \rho (h) dh$$

I don't know how to handle this equation by integrating both members....
 
  • #120
Hak said:
Okay. I get to the equation:

$$T \ d \rho = \left(T_0 B - \frac{g}{R} \right) \rho (h) dh$$

I don't know how to handle this equation by integrating both members....
The method of separation of variables... All factors involving ##\rho## on the left, and everything involving ##h## on the right.

$$ \frac{d \rho}{\rho} = \left(T_0 \beta - \frac{g}{R} \right) \frac{dh}{T(h)} $$
 
  • #121
erobz said:
The method of separation of variables... All factors involving ##\rho## on the left, and everything involving ##h## on the right.

$$ \frac{d \rho}{\rho} = \left(T_0 \beta - \frac{g}{R} \right) \frac{dh}{T(h)} $$
Thanks. I get:

$$ ln |\rho| + c_1 = - \left(T_0 \beta - \frac{g}{R} \right) \frac{ln |1 - \beta h|}{T_0 \beta} + c_2$$, from which:

$$ \rho (h) = e^{- \left(T_0 \beta - \frac{g}{R} \right) \frac{ln |1 - \beta h|}{T_0 \beta} + c}$$.

This is the same result as before, with the difference that I had previously inserted an extra ##h## term, which was wrong.
 
  • #122
Hak said:
Thanks. I get:

$$ ln |\rho| + c_1 = - \left(T_0 \beta - \frac{g}{R} \right) \frac{ln |1 - \beta h|}{T_0 \beta} + c_2$$, from which:

$$ \rho (h) = e^{- \left(T_0 \beta - \frac{g}{R} \right) \frac{ln |1 - \beta h|}{T_0 \beta} + c}$$.

This is the same result as before, with the difference that I had previously inserted an extra ##h## term, which was wrong.

It may be the same, but you are overcomplicating it and making it virtually unrecognizable. Use definite integrals on each side:

$$ \int_{\rho_o}^{\rho(h)} \frac{d \rho}{\rho} =\left(T_0 \beta - \frac{g}{R} \right)
\int_0^h \frac{dh}{T_o - T_o \beta h } $$
 
Last edited:
  • #123
erobz said:
It may be the same, but you are overcomplicating it and making it virtually unrecognizable. Use definite integrals on each side:

$$ \int_{p_o}^{\rho(z)} \frac{d \rho}{\rho} =\left(T_0 \beta - \frac{g}{R} \right)
\int_0^h \frac{dh}{T_o - T_o \beta h } $$
OK. I get:
$$ln |\rho (z)| - ln|\rho_0| = \frac{ln |T_0| - ln |T_0 (1- \beta h)|}{T_0 \beta} \left(T_0 \beta - \frac{g}{R} \right) $$. Right?

Edit. I had forgotten part of the equation.
 
  • #124
Hak said:
OK. I get:
$$ln |\rho (z)| - ln|\rho_0| = \frac{ln |T_0| - ln |T_0 (1- \beta h)|}{T_0 \beta}$$. Right?
Close, but you are forgetting the factor out front, and use log rules to turn differences of logs into a log of a quotient, and then rules about logs and factors, and exponents etc... once complete, exponentiate both sides.
 
  • #125
erobz said:
Close, but you are forgetting the factor out front, and use log rules to turn differences into quotients, and logs and factors.
OK. I get:

$$ln |\frac{\rho(z)}{\rho_0}| = - ln |1 - T_0 \beta| \left(1 - \frac{g}{T_0 \beta R} \right) $$, from which:

$$\rho (z) = \rho_0 \cdot e ^ {- ln |1 - T_0 \beta| \left(1 - \frac{g}{T_0 \beta R} \right)}$$.
Right?
 
  • #126
Hak said:
OK. I get:

$$ln |\frac{\rho(z)}{\rho_0}| = - ln |1 - T_0 \beta| \left(1 - \frac{g}{T_0 \beta R} \right) $$, from which:
Try again. The difference of logs on the RHS, where did it go?

Then...Rules about logs: specifically ##a ln(x) = ln(x^a)##

Only then do you exponentiate both sides.
 
  • #127
erobz said:
Try again. The difference of logs on the RHS, where did it go?
It is correct, or not? We have:

$$ln |T_0| - ln |T_0 (1 - \beta h)| \Rightarrow \ ln |\frac{T_0}{T_0 (1 - \beta h)}| = ln |\frac{1}{(1 - \beta h)}| = ln |(1 - \beta h)|^{-1} = - ln |(1 - \beta h)| $$.

Where is the mistake?
 
  • #128
Hak said:
It is correct, or not? We have:

$$ln |T_0| - ln |T_0 (1 - \beta h)| \Rightarrow \ ln |\frac{T_0}{T_0 (1 - \beta h)}| = ln |\frac{1}{(1 - \beta h)}| = ln |(1 - \beta h)|^{-1} = - ln |(1 - \beta h)| $$.

Where is the mistake?
Sorry, Thats fine. I missed that you factored out ##T_o##.

But also apply log power rule to the factor out front to bring it inside the logarithm as an exponent. You want to be ending with this form, before exponentiation:

$$\ln (a) = \ln(b^c) $$
 
  • #129
erobz said:
But also apply log power rule to the factor out front to bring it inside the logarithm
OK, I get:

$$ln |(1- \beta h) ^ {(1 - \frac{g}{T_0 \beta R})}|$$.

Right?
 
  • #130
Hak said:
OK, I get:

$$ln |(1- T_0 \beta) ^ {(1 - \frac{g}{T_0 \beta R})}|$$.

Right?
Yeah, for the right hand side. Then exponentiate both sides.
 
  • #131
I get:
$$\rho (z) = \rho_0 |(1- \beta h) ^ {(1 - \frac{g}{T_0 \beta R})}|$$.

Right?
 
  • #132
Hak said:
I get:
$$\rho (z) = \rho_0 |(1- \beta h) ^ {(1 - \frac{g}{T_0 \beta R})}|$$.

Right?
I get the negative of that exponent. ## \left( \frac{g}{RT_o \beta} - 1 \right)##
 
  • #133
erobz said:
I get the negative of that exponent. ## \frac{g}{RT_o \beta - 1}##
Yes, you are right, I forgot to include it.
 
  • #134
$$\rho (z) = \rho_0 |(1- \beta h) ^ {(\frac{g}{T_0 \beta R} - 1)}|$$ is the right expression.
 
  • #135
Now, what should be done?
 
  • #136
Hak said:
$$\rho (z) = \rho_0 |(1- \beta h) ^ {(\frac{g}{T_0 \beta R} - 1)}|$$ is the right expression.
you don't need the absolute values
 
  • #137
erobz said:
you don't need the absolute values
OK, thanks. Now?
 
  • #138
Hak said:
OK, thanks. Now?
Now, it should be clear that the function given for ##\rho## in the problem statement does not strictly adhere to the ideal gas law assumption that is used to solve the problem. Using ideal gas law, we see a power law. In the problem it was linear.
 
  • #139
erobz said:
Now, it should be clear that the function given for ##\rho## in the problem statement does not strictly adhere to the ideal gas law assumption that is used to solve the problem. Using ideal gas law, we see a power law. In the problem it was linear.
Right, thank you. What conclusions can be drawn from this?
 
  • #140
Hak said:
Right, thank you. What conclusions can be drawn from this?
I don't know, but it could be that the linear function approximates the power law reasonably well over these altitudes, or they simply chose a function for ##\rho## which makes the problem solvable algebraically. Yet another interpretation is that the linear function is empirical, and its the ideal gas law that was the issue. You have to plot the functions to see why they might have done what they have done.
 

Similar threads

  • Introductory Physics Homework Help
Replies
4
Views
1K
  • Introductory Physics Homework Help
Replies
1
Views
1K
  • Introductory Physics Homework Help
Replies
6
Views
1K
Replies
1
Views
736
  • Introductory Physics Homework Help
Replies
2
Views
653
  • Introductory Physics Homework Help
Replies
13
Views
7K
  • High Energy, Nuclear, Particle Physics
Replies
6
Views
2K
  • Advanced Physics Homework Help
Replies
0
Views
684
  • Introductory Physics Homework Help
Replies
1
Views
1K
  • Introductory Physics Homework Help
Replies
21
Views
2K
Back
Top